Đến nội dung

Nguyen Xuan Hieu nội dung

Có 39 mục bởi Nguyen Xuan Hieu (Tìm giới hạn từ 07-06-2020)



Sắp theo                Sắp xếp  

#677911 Topic BẤT ĐẲNG THỨC ôn thi vào lớp 10 THPT 2017 - 2018

Đã gửi bởi Nguyen Xuan Hieu on 18-04-2017 - 20:50 trong Bất đẳng thức và cực trị

Bài 15:(Nguyen Xuan Hieu)
Cho $a,b,c>0$.Chứng minh rằng:
$7(a^3+b^3+c^3)+6 \geq 3\sqrt{ac}(2ac+1)+3\sqrt{bc}(2bc+1)+3\sqrt{ab}(2ab+1)$
 




#678073 Topic BẤT ĐẲNG THỨC ôn thi vào lớp 10 THPT 2017 - 2018

Đã gửi bởi Nguyen Xuan Hieu on 20-04-2017 - 05:01 trong Bất đẳng thức và cực trị

Lời giải bài 17:

Ta có các bất đẳng thức sau:

         $3=a+b+c\geq 3\sqrt[3]{abc} \Rightarrow abc\leq 1 $

         

          $9=(a+b+c)^{2}\geq 3(ab+bc+ca) \Rightarrow ab+bc+ca\leq 3$

       

          $3\sqrt[3]{a^{2}b^{2}}(3\sqrt[3]{c^{2}}+1)= 9\sqrt[3]{a^{2}b^{2}c^{2}}+3\sqrt[3]{a^{2}b^{2}} \leq [3(a^{2}b^{2}c^{2}+1+1)]+                   (a^{2}b^{2}+1+1)$     $(AM-GM)$

 

           $\Rightarrow    3\sqrt[3]{a^{2}b^{2}}(3\sqrt[3]{c^{2}}+1)-8 \leq 3a^{2}b^{2}c^{2}+a^{2}b^{2}$

 

           $\Rightarrow \frac{a^{2}+1}{3\sqrt[3]{a^{2}b^{2}}(3\sqrt[3]{c^{2}}+1)-8} \geq \frac{a^{2}+1}{3a^{2}b^{2}c^{2}+a^{2}b^{2}}$

 

           $\Rightarrow \sum_{cyc}\frac{a^{2}+1}{3\sqrt[3]{a^{2}b^{2}}(3\sqrt[3]{c^{2}}+1)-8} \geq \sum_{cyc}\frac{a^{2}+1}         {3a^{2}b^{2}c^{2}+a^{2}b^{2}}$

 

Mà      $ \sum_{cyc}\frac{a^{2}+1}{3a^{2}b^{2}c^{2}+a^{2}b^{2}} \geq \sum_{cyc}\frac{(a+1)^{2}}{6a^{2}b^{2}c^{2}+2a^{2}b^{2}} \geq$

           $\frac{(a+b+c+1+1+1)^{2}}{18a^{2}b^{2}c^{2}+2(a^{2}b^{2}+b^{2}c^{2}+c^{2}a^{2})}= \frac{18}{   9a^{2}b^{2}c^{2}+a^{2}b^{2}+b^{2}c^{2}+c^{2}a^{2}}$  $(1)$

Mặt khác 

                       $9a^{2}b^{2}c^{2}+a^{2}b^{2}+b^{2}c^{2}+c^{2}a^{2} \leq 9abc.1 + a^{2}b^{2}+b^{2}c^{2}+c^{2}a^{2}$

                        $= 2abc(a+b+c)+ a^{2}b^{2}+b^{2}c^{2}+c^{2}a^{2} + 3abc = (ab+bc+ca)^{2}+3abc \leq 3^{2}+3= 12$

                       $\Rightarrow 9a^{2}b^{2}c^{2}+a^{2}b^{2}+b^{2}c^{2}+c^{2}a^{2} \leq 12$  $(2)$

Từ $(1)$ và $(2)$ thì $\sum_{cyc}\frac{a^{2}+1}{3a^{2}b^{2}c^{2}+a^{2}b^{2}} \geq \frac{3}{2}$

                                $\Rightarrow \sum_{cyc}\frac{a^{2}+1}{3\sqrt[3]{a^{2}b^{2}}(3\sqrt[3]{c^{2}}+1)-8} \geq \frac{3}{2}$  $(Q.E.D)$

Cách khác ngắn gọn hơn:
Điều phải chứng minh:
$\sum_{cyc} \dfrac{2(a^2+1)}{3\sqrt[3]{a^2b^2}(1+3\sqrt[3]{c^2})-8}\geq 3$.
Ta có:
$\sum_{cyc} \dfrac{2(a^2+1)}{3\sqrt[3]{a^2b^2}(1+3\sqrt[3]{c^2})-8}
\\=\sum_{cyc} \dfrac{a^2+a^2+1+1}{3\sqrt[3]{a^2b^2}+9\sqrt[3]{a^2b^2c^2}-8}
\\\geq \sum_{cyc} \dfrac{a^2+2a+1}{a+b+ab+3(ab+bc+ca)-8}
\\\geq \sum_{cyc} \dfrac{a^2+2a+1}{a+b+ab+(a+b+c)^2-8}
\\=\sum_{cyc} \dfrac{(a+1)^2}{(a+1)(b+1)}
\\=\sum_{cyc} \dfrac{(a+1)}{(b+1)}
\\\geq 3$.

 




#677983 Topic BẤT ĐẲNG THỨC ôn thi vào lớp 10 THPT 2017 - 2018

Đã gửi bởi Nguyen Xuan Hieu on 19-04-2017 - 05:54 trong Bất đẳng thức và cực trị

Bài 17:(Nguyen Xuan Hieu)
Cho $a,b,c$ là các số thực dương thõa mãn  $a+b+c=3$.Chứng minh rằng:
$\sum_{cyc} \dfrac{a^2+1}{3\sqrt[3]{a^2b^2}(1+3\sqrt[3]{c^2})-8} \geq \dfrac{3}{2}$
 




#683216 Topic ôn thi hình học vào cấp 3 chuyên

Đã gửi bởi Nguyen Xuan Hieu on 05-06-2017 - 15:25 trong Hình học

Bài 108:Cho 2 đường tròn (O,R) và (O',R') cắt nhau tại A và B. Trên tia đối của AB lấy M, từ M vẽ các tiếp tuyến MC,MD với (O') và D thuộc miền trong của (O). AC, AD cắt (O) tại P,Q chứng minh P và Q luôn đi qua một điểm cố định.




#708528 [TOPIC] HÌNH HỌC ÔN THI VÀO THPT CHUYÊN 2018-2019

Đã gửi bởi Nguyen Xuan Hieu on 16-05-2018 - 14:58 trong Tài liệu - Đề thi

Bài 53. Cho đường tròn tâm O. Từ điểm M nằm ngoài đường tròn vẽ tiếp tuyến MA, MC (A, C là tiếp điểm), B thuộc cung lớn AC sao cho MB nằm giữa MO và MC. Tia MB cắt đường tròn tại Q khác B, cắt CA tại N.

        a) Gọi T là trung điểm của BQ. Chứng minh rằng MQ.MB=MN.MT

        b) Gọi K là điểm đối xứng với C qua B. Qua Q kẻ đường thẳng song song với BC cắt CM tại H. Chứng minh rằng QH, AC, MK đồng quy.

a)$MQ.MB=MA^2$(Do tam giác $MAQ$ đồng dạng tam giác $MBA$)

Mặt khác do $T$ là trung điểm $BQ$ nên $\widehat{OTQ}=90^0$. Từ đó suy ra 5 điểm $M,A,T,O,C$ thuộc một đường tròn.

$\Rightarrow \widehat{MAC}=\widehat{MCA}=\widehat{ATM}$

Từ đó $\Delta MAN \sim \Delta MTA \Rightarrow MN.MT=MA^2$

Do đó $MN.MT=MQ.MB$

b)Tứ giác $AQCB$ điều hòa

$\Rightarrow QB.AC=2AB.QC(*)$
Gọi $W$ là giao điểm của $AC$ và $MK$

$\Rightarrow Q$ là trung điểm của $HW$(Do $HW \parallel CK$ và $BC=BK$)

Gọi $W'$ là giao điểm của $AC$ và $HQ$

$\Delta HQC \sim \Delta QCB \Rightarrow HQ/HC=QC/QB$

$\Delta HCW' \sim \Delta ABC \Rightarrow HW'/HC=AC/AB$

Do (*) $\Rightarrow HW'=2HQ$

Do đó $Q$ là trung điểm $HW'$

Từ đó suy ra $W' \equiv W$(DPCM)
Hình gửi kèm32549691_2057605344510326_50235186255757
 




#683396 Đề thi chuyên toán tỉnh Thái Bình 2017 - 2018

Đã gửi bởi Nguyen Xuan Hieu on 06-06-2017 - 19:43 trong Tài liệu - Đề thi

Câu bất mình làm hơi trầy cối!

Theo $Holder$ ta có: $${\left( {\sum\limits_{cyc} {\frac{1}{{a\sqrt {3a + 2b} }}} } \right)^2}\left( {\sum\limits_{cyc} {\frac{{3a + 2b}}{a}} } \right) \geqslant {\left( {\frac{1}{a} + \frac{1}{b} + \frac{1}{c}} \right)^3}$$Khi đó bất đẳng thức cần chứng minh tương đương $$\frac{{{{\left( {\frac{1}{a} + \frac{1}{b} + \frac{1}{c}} \right)}^3}}}{{9 + 2\sum\limits_{cyc} {\frac{b}{a}} }} \geqslant \frac{9}{{5abc}} \Leftrightarrow {\left( {\frac{1}{a} + \frac{1}{b} + \frac{1}{c}} \right)^3} \geqslant \frac{{81}}{{5abc}} + \frac{{18}}{5}\left( {\frac{1}{{{a^2}c}} + \frac{1}{{{b^2}a}} + \frac{1}{{{c^2}b}}} \right)$$Đặt $\left( {\frac{1}{a};\frac{1}{b};\frac{1}{c}} \right) \to \left( {x,y,z} \right)$ bất đẳng thức trở thành: $${\left( {x + y + z} \right)^3} \geqslant \frac{{81}}{5}xyz + \frac{{18}}{5}\left( {{x^2}z + {y^2}x + {z^2}y} \right)$$Không mất tính tổng quát ta giả sử $c = \min \left\{ {a,b,c} \right\}$. Ta có: $${\left( {x + y + z} \right)^3} - \frac{{81}}{5}xyz - \frac{{18}}{5}\left( {{x^2}z + {y^2}x + {z^2}y} \right)$$$$ = {\left( {x - y} \right)^2}\left( {x + y + \frac{{17}}{5}z} \right) + \left( {x - z} \right)\left( {y - z} \right)\left( {4x + \frac{2}{5}y + z} \right) \geqslant 0$$Vậy bất đẳng thức được chứng minh thành công!

Ặc!! Chắc chết :v




#683518 Đề thi chuyên toán tỉnh Thái Bình 2017 - 2018

Đã gửi bởi Nguyen Xuan Hieu on 07-06-2017 - 13:59 trong Tài liệu - Đề thi

SỞ GIÁO DỤC VÀ ĐÀO TẠO                                                    ĐỀ THI TUYỂN SINH VÀO 10 CHUYÊN THÁI BÌNH

             THÁI BÌNH                                                                                               NĂM 2017-2018

                                                                                                                   MÔN THI: TOÁN CHUYÊN

                                            THỜI GIAN: 150'

 

 

 

Câu 1:(2 điểm) 

 

1. Cho $a,b,$ thực.Chứng minh ít nhất 1 trong 2 phương trình sau vô nghiệm:

 

$x^{2}+2ax+2a^{2}-b^{2}+1=0$ (1)

$x^{2}+2bx+3b^{2}-ab=0$ (2)

 

2. Cho $x,y,z$ thực sao cho $\left\{\begin{matrix} x+y+z=0 & \\ xyz\neq 0 & \end{matrix}\right.$

Tính:

$P=\frac{x^{2}}{-x^{2}+y^{2}+z^{2}}+\frac{y^{2}}{x^{2}-y^{2}+z^{2}}+\frac{z^{2}}{x^{2}+y^{2}-z^{2}}$

 

Câu 2:(2,5 điểm)

 

1.Giải phương trình:

 

$\sqrt{x^{2}+4x+12}=2x-4+\sqrt{x+1}$

 

2.Giải hệ:

 

$\left\{\begin{matrix} x^{3}+y^{3}-4xy\left ( \frac{2}{x-y}-1 \right ) &=4\left ( 4+xy \right ) \\ \sqrt{x-y}+3\sqrt{2y^{3}-y+1} &=2y^{3}-x+3 \end{matrix}\right.$

 

Câu 3:(1 điểm) Tìm $x,y$ nguyên thỏa mãn:

 

$x^{3}-y^{3}=6xy+3$

 

Câu 4:(3 điểm) Cho $ABCD$ nội tiếp $(O)$.$BA\cap CD\equiv E;AD\cap BC\equiv F$.$M,N$ là trung điểm $AC,BD$.Phân giác trong $\widehat{BEC},\widehat{BFA}$ cắt nhau tại $K$.Chứng minh:

1.$\widehat{DEF}+\widehat{DFE}=\widehat{ABC}$ và $\triangle EKF$ vuông.

2.$EM.BD=EN.AC$

3. $K,M,N$ thẳng hàng.

 

Câu 5:(1,5 điểm)

1.Cho $a,b,c$ thực dương.Chứng minh:

 

$\frac{1}{a\sqrt{3a+2b}}+\frac{1}{b\sqrt{3b+2c}}+\frac{1}{c\sqrt{3c+2a}}\geq \frac{3}{\sqrt{5abc}}$

 

2.Cho 5 số tự nhiên phân biệt sao cho tổng 3 số bất kỳ trong chúng $>$ tổng 2 số còn lại.Chứng minh 5 số đã cho không nhỏ hơn 5.

 

 

 

 

 

 

 

 

P/s: Đề này khá dễ!

 

bạn xem phân tích có nhầm k, đề là $x^3+y^3$ mà?

Câu hệ phương trình cái phương trình $(1)$ là $x^2+y^2$ nhé ._. không phải là $x^3+y^3$ nhé




#683373 Đề thi chuyên toán tỉnh Thái Bình 2017 - 2018

Đã gửi bởi Nguyen Xuan Hieu on 06-06-2017 - 17:02 trong Tài liệu - Đề thi

Câu tổ:
Gọi $5$ số tự nhiên phân biệt đó lần lượt là: $a_1,a_2,a_3,a_4,a_5$ và $a_1<a_2<a_3<a_4<a_5$
Giả sử tồn tại một số $a_1$ nhỏ hơn $5$ và 4 số còn lại lớn hơn hoặc bằng $5$.
Khi đó:$a_1+a_2+a_3 \leq 4+a_4-2+a_5-2=a_4+a_5$.
Trái với giả thuyết là tổng 3 số luôn lớn hơn 3 số còn lại.
Trường hợp tồn tại 2 số $a_1,a_2$ nhỏ hơn $5$ và 3 số còn lại lớn hơn bằng $5$.
$a_1+a_2+a_3 \leq 4+3+a_3=5+2+a_3 \leq a_4+2+a_5-2=a_4+a_5$(Vô lý)
Tương tự với các trường hợp còn lại...




#683500 Đề thi chuyên toán tỉnh Thái Bình 2017 - 2018

Đã gửi bởi Nguyen Xuan Hieu on 07-06-2017 - 11:46 trong Tài liệu - Đề thi

Câu c hình chứng minh sao thế nhỉ ._. Bác nào chỉ em cái :v




#683412 Đề thi chuyên toán tỉnh Thái Bình 2017 - 2018

Đã gửi bởi Nguyen Xuan Hieu on 06-06-2017 - 20:57 trong Tài liệu - Đề thi

haha mk nói là cách trầy cối mà, bài này có thể giải đơn giản bằng cách đổi biến về $\left( {\frac{1}{a};\frac{1}{b};\frac{1}{c}} \right) \to \left( {x,y,z} \right)$ sau đó dùng $Cauchy - Schwarz$ đánh giá mẫu hoặc dùng $AM-GM$: $x + y + z \geqslant 3\root 3 \of {xyz} $ sau đó biến đổi tương đương nó cũng ra đáp án :D  :icon6:

Ok. Nãy thấy trên fb gồi :V mà cái người chụp ảnh màn hình trên fb là bạn hả =))




#683374 Đề thi chuyên toán tỉnh Thái Bình 2017 - 2018

Đã gửi bởi Nguyen Xuan Hieu on 06-06-2017 - 17:07 trong Tài liệu - Đề thi

Câu hệ phương trình biến đổi thành:
$\left\{\begin{matrix}
 &\dfrac{(x-y-4)(x^2+4x+y^2-4y)}{x-y}=0 \\
 &(\sqrt{x-y}-\sqrt{2y^2-y+1}+2)(\sqrt{x-y}+\sqrt{2y^2-y+1}-1)=0
\end{matrix}\right.$
...
 




#683367 Đề thi chuyên toán tỉnh Thái Bình 2017 - 2018

Đã gửi bởi Nguyen Xuan Hieu on 06-06-2017 - 16:23 trong Tài liệu - Đề thi

Câu 1:
1)Giả sử cả hai phương trình đều có nghiệm khi đó denta của cả hai phương trình phải lớn hơn hoặc bằng 0:
$\Delta'_1=a^2-2a^2+b^2-1=-a^2+b^2-1 \geq 0
\\\Delta'_2=b^2-3b^2+ab=-2b^2+ab \geq 0
\\\Leftrightarrow  \Delta'_1+\Delta'_2=-a^2-b^2+ab-1 \geq 0
\\\Leftrightarrow -2a^2-2b^2+2ab-2 \geq 0
\\\Leftrightarrow -(a-b)^2-a^2-b^2-2 \geq 0$
Điều này hiển nhiên vô lý do $(a-b)^2,a^2,b^2 \geq 0$
Do đó tồn tại ít nhất một phương trình vô nghiệm.
2)$\dfrac{x^2}{y^2+z^2-x^2}
\\=\dfrac{x^2}{(y-x)(y+x)+z^2}
\\=\dfrac{x^2}{(y-x).(-z)+z^2}
\\=\dfrac{x^2}{z(x+y+z)-2zy}
\\=\dfrac{x^2}{-2zy}$
Tương tự cộng vế lại quy đồng($x,y,z \neq 0$) lên ta sẽ có:
$P=\dfrac{-1}{2}(\dfrac{x^3+y^3+z^3}{xyz})
\\P=\dfrac{-1}{2}(\dfrac{x^3+y^3+z^3-3xyz}{xyz}+3)
\\P=\dfrac{-1}{2}[\dfrac{(x+y+z)(x^2+y^2+z^2-xy-yz-zx)}{xyz}+3]
\\P=\dfrac{-3}{2}$



#683196 Đề thi toán chuyên - chuyên KHTN ĐHQG HÀ Nội vòng 2 2017

Đã gửi bởi Nguyen Xuan Hieu on 05-06-2017 - 11:52 trong Tài liệu - Đề thi

Câu II 2.

Từ giả thiết ta có được $\frac{1}{(a+1)(b+1)}+\frac{1}{(c+1)(b+1)}+\frac{1}{(a+1)(c+1)}=1$

 Đặt $a+1=\frac{\sqrt{3}}{x}, b+1=\frac{\sqrt{3}}{y},c+1=\frac{\sqrt{3}}{z}$

Giả thiết trở thành $xy+yz+zx=3$ và

$P= \sqrt{3} ( \frac{1}{\frac{3}{x}+x} +\frac{1}{\frac{3}{y}+y} +\frac{1}{\frac{3}{z}+z})$

   $= \sqrt{3} (\frac{x}{x^{2}+3}+\frac{y}{y^{2}+3}+\frac{z}{z^{2}+3})$

Sử dụng giả thiết ta có

  $P=\sqrt{3}( \frac{x}{(x+y)(x+z)}+ \frac{y}{(x+y)(y+z)}+ \frac{z}{(z+y)(x+z)})$

    $=\sqrt{3}( \frac{2(xy+yz+zx)}{(x+y)(y+z)(z+x)})$

Mặt khác $(x+y)(y+z)(z+x) \geq \frac{8}{9}(x+y+z)(xy+yz+zx) \geq \frac{8}{3}(xy+yz+zx)$

Suy ra $P \leq \sqrt{3}\frac{3}{4}= \frac{3\sqrt{3}}{4}$

Đẳng thức xảy ra khi $a=b=c = \sqrt{3}-1$                                                      

Đặt $a+1=\dfrac{1}{x}$ được không ạ?




#683193 Đề thi toán chuyên - chuyên KHTN ĐHQG HÀ Nội vòng 2 2017

Đã gửi bởi Nguyen Xuan Hieu on 05-06-2017 - 11:36 trong Tài liệu - Đề thi

Câu 2.1:
a)$p(p-1)=q(q^2-1)$
Dễ thấy $p,q$ là số nguyên tố nên :$p|(q^2-1)$ đặt $q^2-1=kp,k \in N$.
Thay vào sẽ được $p-1=kq$.
Do đó tồn tại số $k$...(dpcm)
b)Ta có:$p|(q+1)$ nên $q+1 \geq p=kq+1 \geq q+1$.
Dấu '=' khi $q+1=p$ thay vào giải ra $p,q$




#683921 Đề thi toán vòng 2 thpt chuyên Lê Hồng Phong Nam Định 2017-2018

Đã gửi bởi Nguyen Xuan Hieu on 10-06-2017 - 15:30 trong Tài liệu - Đề thi

Bài 3:

a) $\widehat{DIC}=\widehat{BOA}=\widehat{BEC}$

$\Rightarrow Q.E.D$

 

 

b) Hình như phải là $\widehat{DHB}=\frac{1}{2}\widehat{DHE}$

Ta có:$AD.AE=AB^{2}=AH.AO$

Do đó,$DHOE$ nội tiếp

$\Rightarrow \widehat{DHA}=\widehat{DEO}=\widehat{ODE}=\widehat{OHE}\\\Rightarrow \widehat{DHB}=\frac{1}{2}\widehat{DHE}\Rightarrow Q.E.D$

 

 

c) Tính chất tứ giác điều hòa

Đề đúng rồi đấy.

Gọi $J$ là trung điểm của $DE$.
Khi đó dễ thấy $OJ \perp AE$ nên $5$ điểm: $A,B,J,O,C,A$ cùng thuộc một đường tròn.
Khi đó tứ giác $JBAC$ nội tiếp $\Rightarrow \widehat{DJC}=\widehat{AJC}=\widehat{ABC}=\widehat{BIF}=\widehat{DIC}$.
Do đó tứ giác $DIJC$ nội tiếp $\Rightarrow \widehat{DJC}=\widehat{DCI}=\widehat{DCB}=\widehat{DEB}$.
Do đó $IJ//EF$ mà $J$ là trung điểm của $DE \Rightarrow I$ là trung điểm của $DF$.
Mặt khác $I$ cũng là trung điểm của $BH$ nên $BDHF$ là hình bình hành.
Lại có:$AD.AE=AB^2=AC^2=AH.AO$ do đó,$DHOE$ nội tiếp
$\Rightarrow \widehat{DHA}=\widehat{DEO}=\widehat{ODE}=\widehat{OHE}\\\Rightarrow \widehat{DHB}=\frac{1}{2}\widehat{DHE}$
Từ điều trên dễ thấy $\widehat{EBH}=\widehat{EHB}$ nên tam giác $EBH$ cân mà $I$ là trung điểm $BH$ nên $KI \perp CI$ mà $EFIC$ nội tiếp nên $CF \perp FE$.
Mà $H$ là trung điểm $BC$.
Nên $\widehat{HFC}=\widehat{HCF}$ hay $\widehat{BHF}=2\widehat{HCF}$.
Mà $\widehat{DBC}=\widehat{BHF}(BD//FH) \Rightarrow \widehat{DBC}=2\widehat{HCF}$.
Dễ dàng chứng minh tứ giác $FHCK$ nt ($\widehat{BHF}=\widehat{DBC}=\widehat{DKC}$) nên $\widehat{HCF}=\widehat{FKH}=\widehat{DKH}$.
Do đó $\widehat{BHF}=2\widehat{HCF}=2\widehat{DKH}$
P/s: Cho hỏi sao up ảnh lên diễn đàn được vậy? Mình chụp màn hình rồi crt +v thì nó bảo là không hỗ trợ định dạng

 



#683834 Đề tuyển sinh vào 10 THPT chuyên Phan Bội Châu - Nghệ An 2017-2018

Đã gửi bởi Nguyen Xuan Hieu on 09-06-2017 - 19:40 trong Tài liệu - Đề thi

Ba

 

Loại nghiệm $x=\frac{-20}{3}$ thì giải $2\sqrt(x+4)+ \sqrt(x-4)=7$

 cũng ra x=5 thôi bạn, có gì sai đâu

Hix. Xin lỗi tui chả để ý :v




#682657 Đề thi tuyển sinh vào THPT Chuyên Sư Phạm 2017 vòng 1 + vòng 2

Đã gửi bởi Nguyen Xuan Hieu on 01-06-2017 - 20:11 trong Tài liệu - Đề thi

Câu 1:
Giả sử trong 4 số đều bé hơn $3$.Đặt tổng 4 số đó là $P$.Khi đó $P<3.4=12$.(*)
Ta có:$P=a^2+b^2+c^2+d^2+2(\sum \dfrac{1}{a}) \geq \dfrac{(a+b+c+d)^2}{4}+\dfrac{16}{a+b+c+d}+\dfrac{16}{a+b+c+d}$.
Sau đó Cauchy 3 số thì sẽ được $P \geq 12$.Trái với (*) do đó ta có đpcm.
 




#683876 Đề tuyển sinh vào 10 THPT chuyên Phan Bội Châu - Nghệ An 2017-2018

Đã gửi bởi Nguyen Xuan Hieu on 10-06-2017 - 08:22 trong Tài liệu - Đề thi

Bác nào giải câu tổ cho em tham khảo với :v




#683788 Đề tuyển sinh vào 10 THPT chuyên Phan Bội Châu - Nghệ An 2017-2018

Đã gửi bởi Nguyen Xuan Hieu on 09-06-2017 - 14:32 trong Tài liệu - Đề thi

Câu 1a)
Phương trình tương đương:
$3(x-5)+7(\sqrt{x-4}-1)=14(\sqrt{x+4}-3) \\\Leftrightarrow 3(x-5)+\dfrac{7(x-5)}{\sqrt{x-4}+1}=\dfrac{14(x-5)}{\sqrt{x+4}+3}$.
Tới đây xét $x-5=0 \\\Rightarrow x=5$.
Xét $x \neq 5$ thì $3+\dfrac{7}{\sqrt{x-4}+1}=\dfrac{14}{\sqrt{x+4}+3}$.
Dễ thấy $VT<3,VP>3$ với điều kiện của $x$ do đó $x=5$ là nghiệm duy nhất.




#683766 Đề tuyển sinh vào 10 THPT chuyên Phan Bội Châu - Nghệ An 2017-2018

Đã gửi bởi Nguyen Xuan Hieu on 09-06-2017 - 11:11 trong Tài liệu - Đề thi

Câu 2:
$S(n)=n^2-2017n+10=n(n-2017)+10$.
Dễ thấy nếu $n<2017$ thì $S(n)<0$.
$n=2017$ thì $S(n)=10$ thõa mãn.
$n \geq 2018$ thì $S(n)=n^2-2017n+10>n$.
Dễ thấy $n$ lúc này là  số có 4 chữ số trở lên nên không tồn tại $S(n)>n$
Vậy $n=2017$




#683787 Đề tuyển sinh vào 10 THPT chuyên Phan Bội Châu - Nghệ An 2017-2018

Đã gửi bởi Nguyen Xuan Hieu on 09-06-2017 - 14:29 trong Tài liệu - Đề thi

 

Câu 1 a)$$(3x-20)-7(2\sqrt{x+4}-\sqrt{x-4})=0 \Leftrightarrow (3x-20)-\frac{7}{2\sqrt{x+4}+\sqrt{x-4}}(3x-20)\\ = (3x-20)(1-\frac{7}{2\sqrt{x+4}+ \sqrt{x-4}})$$
b) Đặt x+1=a
y+1=b
Đưa về hệ đối xứng $\left\{\begin{matrix} 6a+4b=a^2\\ 6b+4a=b^2 \end{matrix}\right.

Câu 1a sai rồi. Phải có nhân tử là $x-5$. Tách nhân liên hợp là ra.
P/s: Bác nào full bất, tổ cái :v. Tý em gõ hình cho :v




#683116 Đề tuyển sinh vào lớp 10 THPT chuyên Lê Quý Đôn - Quảng Trị năm học 2017-2018

Đã gửi bởi Nguyen Xuan Hieu on 04-06-2017 - 21:09 trong Tài liệu - Đề thi

Câu 3:
a)$DKXD: -2 \leq x \leq 3
\\2\sqrt{3-x}+\sqrt{2+x}=5
\\VT=\sqrt{4(3-x)}+\sqrt{1(2+x)} \leq \dfrac{4+3-x+1+2+x}{2}=5=VP$.
Dấu '=' khi $x=1$.
b)Áp đụng đẳng thức quen thuộc :$a^3+b^3+c^3-3abc=(a+b+c)(a^2+b^2+c^2-ab-bc-ca)$.
Áp dụng $với a=x,b=y,c=1$ khi đó:$x^3+y^3+1-3xy=0 \\\Leftrightarrow (x+y+1)(x^2+y^2+1-xy-x-y)=0$.
Tới đây $x+y+1=0$ thay vào cái dưới rồi tìm $x,y$.
$x^2+y^2+1-xy-x-y=0 \Rightarrow 2x^2+2y^2+2-2xy-2x-2y=0 \\\Rightarrow (x-1)^2+(y-1)^2+(x-y)^2=0 \\\Rightarrow x=y=1$.
Thay vào phương trình dưới thì thỏa mãn.Kết luận:.....




#683115 Đề tuyển sinh vào lớp 10 THPT chuyên Lê Quý Đôn - Quảng Trị năm học 2017-2018

Đã gửi bởi Nguyen Xuan Hieu on 04-06-2017 - 21:01 trong Tài liệu - Đề thi

Câu 4b) Nghiệm nguyên: $7(x+y)=3(x^2+xy+y^2)$.
Xét $(x,y)=(0,0)$ thõa mãn.
Xét $x,y \neq 0$ khi đó:$\dfrac{x^2+xy+y^2}{x+y}=\dfrac{7}{3}$.
Đặt $x^2+xy+y^2=7m,x+y=3m,m \in \mathbb{N}^*$.Do $7m=x^2+xy+y^2>0$.
Từ điều trên dễ tính được $9m^2-7m=xy,x+y=3m$.Do đó $x,y$ là nghiệm của pt $X^2-3mX+9m^2-7m=0$.
Xét điều kiện denta lớn hơn hoặc bằng 0 tìm được $m=0,m=1$.
Với $m=0$ thì vô lý do $x,y \neq 0$.
Với $m=1$ giải ra được $(x,y)=(1,2);(2,1)$.
Kết luận $(x,y)=(0,0);(1,2);(2,1)$
Câu 4a)Dễ thấy số có tận cùng là $7$ khi mũ $4$ lên có tận cùng luôn là $1$.Mà $6^4 \vdots 4$.Vậy chữ số tận cùng là $1$.
 




#683358 Đề thi THPT chuyên Lê Quý Đôn Đà Nẵng 2017

Đã gửi bởi Nguyen Xuan Hieu on 06-06-2017 - 14:17 trong Tài liệu - Đề thi

Lời giải Bài 6

18835760_1912742219008996_47107785253807

Cách nhanh hơn:
Từ phương trình dễ thấy $z$ chẵn.
Đặt $(3z-1)(z+1)=d$.
Khi đó$3(z+1)-(3z-1) \vdots \Rightarrow 4 \vdots d$.
Mà $z+1$ là số lẻ do đó $d=1$.
Mặt khác:$3z-1$ không chia hết cho $3$.
nên đặt $3z-1=5^x$ và $z+1=3^y$
Làm tương tự ...

 




#683912 Đề thi vào lớp 10 chuyên Toán THPT Chuyên Long An 2017 - 2018

Đã gửi bởi Nguyen Xuan Hieu on 10-06-2017 - 14:44 trong Tài liệu - Đề thi

Câu 6:
Tách ra thành: $\dfrac{9x}{2-x}+\dfrac{2-x}{x}+2 \\\geq 6+2=8$.
Dấu '=' khi $x=\dfrac{1}{2}$